LSAT and Law School Admissions Forum

Get expert LSAT preparation and law school admissions advice from PowerScore Test Preparation.

User avatar
 Dave Killoran
PowerScore Staff
  • PowerScore Staff
  • Posts: 5852
  • Joined: Mar 25, 2011
|
#43327
Complete Question Explanation
(The complete setup for this game can be found here: lsat/viewtopic.php?t=16326)

The correct answer choice is (E)

The “if” statement specifies that F must be in group 1, which automatically indicates that J is in group 1 as well. Since the FJ block is in group 1, G, H, and K must be in either group 2 or 3. Looking at the restricted variable H, we can see that H must be in group 3 (note that H has been placed into the “second” space in group 3, signifying that the adult space is yet to be filled). Since H is in group 3, R cannot be in group 3, and we have the following setup:
O94_Game_#2_#7_diagram 1.png
Answer choice (A) is incorrect since group 3 cannot contain three children.

Answer choice (B) is incorrect since R cannot be placed into group 3 with H.

Answer choice (C) is incorrect because J is in group 1.

Answer choice (D) is incorrect because both K and R cannot fit into group 1 along with F and J.

It follows that answer choice (E) is correct.

Surprisingly, this question is correctly answered by only 47% of all test takers. In approaching games in general, there is often some value in bypassing “could be true” questions that appear early in the game. Perhaps a later question will emulate the “F in 1” criterion in question #7 and thus produce a hypothetical which could be used to eliminate wrong answer choices. Regrettably, in this game no such hypothetical appears. However, using this strategy does not cost the test taker any time and so the question could still be completed at the end of the game.
You do not have the required permissions to view the files attached to this post.

Get the most out of your LSAT Prep Plus subscription.

Analyze and track your performance with our Testing and Analytics Package.